Difference between revisions of "1954 AHSME Problems/Problem 35"

 
Line 1: Line 1:
 +
== Problem 35 ==
 +
 +
In the right triangle shown the sum of the distances <math>BM</math> and <math>MA</math> is equal to the sum of the distances <math>BC</math> and <math>CA</math>.
 +
If <math>MB = x, CB = h</math>, and <math>CA = d</math>, then <math>x</math> equals:
 +
 +
<asy>
 +
defaultpen(linewidth(.8pt)+fontsize(10pt));
 +
dotfactor=4;
 +
draw((0,0)--(8,0)--(0,5)--cycle);
 +
label("C",(0,0),SW);
 +
label("A",(8,0),SE);
 +
label("M",(0,5),N);
 +
dot((0,3.5));
 +
label("B",(0,3.5),W);
 +
label("$x$",(0,4.25),W);
 +
label("$h$",(0,1),W);
 +
label("$d$",(4,0),S);</asy>
 +
 +
<math> \textbf{(A)}\ \frac{hd}{2h+d}\qquad\textbf{(B)}\ d-h\qquad\textbf{(C)}\ \frac{1}{2}d\qquad\textbf{(D)}\ h+d-\sqrt{2d}\qquad\textbf{(E)}\ \sqrt{h^2+d^2}-h </math>
 +
 
== Solution 1==
 
== Solution 1==
  

Latest revision as of 18:31, 2 May 2022

Problem 35

In the right triangle shown the sum of the distances $BM$ and $MA$ is equal to the sum of the distances $BC$ and $CA$. If $MB = x, CB = h$, and $CA = d$, then $x$ equals:

[asy] defaultpen(linewidth(.8pt)+fontsize(10pt)); dotfactor=4; draw((0,0)--(8,0)--(0,5)--cycle); label("C",(0,0),SW); label("A",(8,0),SE); label("M",(0,5),N); dot((0,3.5)); label("B",(0,3.5),W); label("$x$",(0,4.25),W); label("$h$",(0,1),W); label("$d$",(4,0),S);[/asy]

$\textbf{(A)}\ \frac{hd}{2h+d}\qquad\textbf{(B)}\ d-h\qquad\textbf{(C)}\ \frac{1}{2}d\qquad\textbf{(D)}\ h+d-\sqrt{2d}\qquad\textbf{(E)}\ \sqrt{h^2+d^2}-h$

Solution 1

The question states that \[h+d = x+\sqrt{(x+h)^2+d^2}\]

We move $x$ to the left: \[h+d-x = \sqrt{(x+h)^2+d^2}\]

We square both sides: \[h^2 + d^2 + x^2 - 2xh - 2xd + 2hd = x^2 + 2xh + h^2 + d^2\]

Cancelling and moving terms, we get: \[4xh + 2xd = 2hd\]

Factoring $x$: \[x(4h+2d) = 2hd\]

Isolating for $x$: \[x=\frac{2hd}{4h+2d}=\frac{hd}{2h+d}\]

Therefore, the answer is $\fbox{A}$


Solution 2

Realize that a 3 - 4 - 5 triangle satisfies these requirements. Checking the answer choices, $\fbox{A}$ is the correct solution.

See Also

1954 AHSC (ProblemsAnswer KeyResources)
Preceded by
Problem 34
Followed by
Problem 36
1 2 3 4 5 6 7 8 9 10 11 12 13 14 15 16 17 18 19 20 21 22 23 24 25 26 27 28 29 30 31 32 33 34 35 36 37 38 39 40 41 42 43 44 45 46 47 48 49 50
All AHSME Problems and Solutions


The problems on this page are copyrighted by the Mathematical Association of America's American Mathematics Competitions. AMC logo.png